\[ \newcommand{\C}{\mathbb{C}} \newcommand{\cont}{\operatorname{\mathsf{C}}} \newcommand{\haar}{\mathsf{m}} \newcommand{\B}{\mathscr{B}} \newcommand{\D}{\mathscr{D}} \newcommand{\P}{\mathcal{P}} \newcommand{\Q}{\mathbb{Q}} \newcommand{\R}{\mathbb{R}} \newcommand{\N}{\mathbb{N}} \newcommand{\Z}{\mathbb{Z}} \newcommand{\sph}{\mathsf{S}^\mathsf{1}} \newcommand{\g}{>} \newcommand{\l}{<} \newcommand{\intd}{\,\mathsf{d}} \newcommand{\Re}{\mathsf{Re}} \newcommand{\area}{\mathop{\mathsf{Area}}} \newcommand{\met}{\mathop{\mathsf{d}}} \newcommand{\emptyset}{\varnothing} \DeclareMathOperator{\borel}{\mathsf{Bor}} \DeclareMathOperator{\baire}{\mathsf{Baire}} \newcommand{\orb}{\mathop{\mathsf{orb}}} \newcommand{\symdiff}{\mathop\triangle} \newcommand{\lp}[1][2]{\operatorname{\mathscr{L}^{\mathsf{#1}}}} \newcommand{\Lp}[1][2]{\operatorname{\mathsf{L}^{\!\mathsf{#1}}}} \]

Week 9 Coursework Test

  1. The matrix $A = [\begin{smallmatrix} 2 & 1 \\ 1 & 1 \end{smallmatrix}]$ has eigenvalues $0 \l \eta \l 1 \l \theta$. Let $v = [\begin{smallmatrix} v_1 \\ v_2 \end{smallmatrix}]$ be an eigenvector of $A$ with eigenvalue $\eta$.
    1. Prove for every $r,s \in \Z$ with $(r,s) \ne (0,0)$ and every $(x,y) \in [0,1)^2$ that \[ \lim_{T \to \infty} \dfrac{1}{T} \int\limits_0^T e^{2 \pi i r(x + tv_1) + 2 \pi i s (y+tv_2)} \intd t = 0 \] by calculating the above Riemann integral and then evaluating the limit.
    2. By analogy with our work on irrational rotations, postulate a uniform distribution statement that would follow from the above result.
    3. Consider a short line segment $L$ in $[0,1)^2$ parallel to $v$. Fix a rectangle $R \subset [0,1)^2$. With \[ T(x,y) = (2x+y \bmod 1, x+y \bmod 1) \] describe what $T^{-n}(L)$ looks like when $n$ is large. Using your postulate, what proportion of $T^{-n}(L)$ do you expect belongs to $R$ as $n$ increases?
  2. Prove the collection \[ \left\{ \left[ \dfrac{a}{10^r}, \dfrac{a+1}{10^r} \right) : r \in \N, 0 \le a \l 10^r \right\} \] of subsets of $[0,1)$ generates the Borel σ-algebra on $[0,1)$.